Paul's Online Notes
Paul's Online Notes
Home / Calculus II / Vectors / Dot Product
Show Mobile Notice Show All Notes Hide All Notes
Mobile Notice
You appear to be on a device with a "narrow" screen width (i.e. you are probably on a mobile phone). Due to the nature of the mathematics on this site it is best views in landscape mode. If your device is not in landscape mode many of the equations will run off the side of your device (should be able to scroll to see them) and some of the menu items will be cut off due to the narrow screen width.

Section 11.3 : Dot Product

5. Determine the angle between \(\vec a = \vec i + 3\vec j - 2\vec k\) and \(\vec b = \left\langle { - 9,1, - 5} \right\rangle \).

Show Solution

Not really a whole lot to do here. All we really need to do is rewrite the formula from the geometric interpretation of the dot product as,

\[\cos \theta = \frac{{\vec a\centerdot \vec b}}{{\left\| {\vec a} \right\|\,\,\left\| {\vec b} \right\|}}\]

This will allow us to quickly determine the angle between the two vectors.

We’ll first need the following quantities (we’ll leave it to you to verify the arithmetic involved in these computations….).

\[\vec a\centerdot \vec b = 4\hspace{0.25in}\hspace{0.25in}\left\| {\vec a} \right\| = \sqrt {14} \hspace{0.25in}\hspace{0.25in}\left\| {\vec b} \right\| = \sqrt {107} \]

The angle between the vectors is then,

\[\cos \theta = \frac{4}{{\sqrt {14} \,\sqrt {107} }} = 0.1033\hspace{0.25in} \Rightarrow \hspace{0.25in}\theta = {\cos ^{ - 1}}\left( {0.1034} \right) = \require{bbox} \bbox[2pt,border:1px solid black]{{1.4673\,\,{\rm{radians}}}}\]